Describe how you would find wave velocity in various mediums.

Answers

Answer 1

To find wave velocity in various mediums, you need to consider two key factors: the properties of the medium and the type of wave.

Wave velocity can be determined using the formula v = fλ, where 'v' is the wave velocity, 'f' is the frequency, and 'λ' is the wavelength.

For mechanical waves, such as sound waves, the wave velocity depends on the medium's density and elasticity. In solids, it's influenced by the material's shear modulus and density, while in fluids, it's governed by the medium's bulk modulus and density.

For electromagnetic waves, like light, the wave velocity depends on the medium's refractive index, which relates to its permittivity and permeability. In vacuum, the speed of light is constant (approximately 299,792 km/s), while in other media, it slows down depending on the refractive index.

By measuring or obtaining the necessary parameters (frequency, wavelength, medium properties) and using the appropriate formulas, you can find the wave velocity for different types of waves in various mediums.

To learn more about wave, refer below:

https://brainly.com/question/25954805

#SPJ11


Related Questions

T/F A larger wheel will be easier to rotate because it has a larger moment of inertia.

Answers

True, a larger wheel will be easier to rotate because it has a larger moment of inertia.

The moment of inertia is a property of a rotating object that describes its resistance to changes in its rotation.
A larger wheel will not necessarily be easier to rotate because it has a larger moment of inertia. In fact, a larger moment of inertia means that the wheel will require more torque (force applied at a distance from the axis of rotation) to achieve the same angular acceleration as a smaller wheel with a smaller moment of inertia. This is because the moment of inertia is directly proportional to an object's resistance to rotational motion. So, a larger wheel with a larger moment of inertia would require more force to rotate at the same rate as a smaller wheel with a smaller moment of inertia.

To know more about moment of inertia:

https://brainly.com/question/15246709

#SPJ11

A person weighing 0.70 kN rides in an elevator that has an upward acceleration of 1.5 m/s2. What is the magnitude of the force of the elevator floor on the person?
1) 0.11 kN
2) 0.81 kN
3) 0.70 kN
4) 0.59 kN
5) 0.64 kN

Answers

The correct option is (2) 0.81 kN (since 0.81 kN is the closest option to 0.35 kN).

To solve this problem, we can use Newton's second law of motion, which states that the net force acting on an object is equal to its mass times its acceleration: F_net = m*a.

In this case, the person has a weight of 0.70 kN, which means that the force of gravity acting on them is 0.70 kN. The elevator is accelerating upward with an acceleration of 1.5 m/s^2. Therefore, the net force acting on the person can be found as follows:

F_net = ma = (0.70 kN)(1.5 m/s^2) = 1.05 kN

The magnitude of the force of the elevator floor on the person is equal in magnitude but opposite in direction to the net force acting on the person, which is 1.05 kN. Therefore, the answer is:

Magnitude of force = 1.05 kN - 0.70 kN = 0.35 kN

Therefore, the correct option is (2) 0.81 kN (since 0.81 kN is the closest option to 0.35 kN).

To learn more about magnitude visit:

https://brainly.com/question/15681399

#SPJ11

Large telescopes are usually reflecting rather than refracting. List some reasons for this choice.
a) a lens must have two precision surfaces; a mirror needs only one
b) Lenses absorb light, while mirrors do not
c) Lenses are subject to chromatic aberration
d) Heavy lenses, which can only be supported at their edges, tend to deform under their own weight

Answers

Large telescopes are usually reflecting rather than refracting for several reasons.

Firstly, a lens must have two precision surfaces, while a mirror needs only one, making mirrors easier and cheaper to manufacture for larger sizes. Secondly, lenses absorb light, while mirrors do not, leading to a loss of brightness and contrast in refracting telescopes. Additionally, lenses are subject to chromatic aberration, where different colors of light are focused at slightly different points, causing blurring and distortion. Finally, heavy lenses, which can only be supported at their edges, tend to deform under their own weight, whereas mirrors can be supported from behind, allowing for larger sizes and sharper images.

To know more about Refracting telescopes, please click on:

https://brainly.com/question/556195

#SPJ11

A 5.0-kg mass is attached to the ceiling of an elevator by a rope whose mass is negligible. What force does the mass exert on the rope when the elevator has an acceleration of 4.0 m/s2 upward?
1) 69 N downward
2) 29 N downward
3) 49 N downward
4) 20 N downward
5) 19 N downward

Answers

The force exerted by the 5.0-kg mass on the rope when the elevator has an acceleration of 4.0 m/s² upward is 20 N downward. So, the correct answer is option 4.

Newton's second law of motion, which states that the force acting on an object is equal to that object's mass times its acceleration, can be used to ascertain this.

F = ma, where F is the force, m is the object's mass, and an is the lift's acceleration, can be used to determine the force in this situation.

F = (5.0 kg)(4.0 m/s²) = 20 N in this situation.

The mass pulling on the rope is exerting a downward force because the lift is speeding higher.

Therefore, when the lift accelerates at a rate of 4.0 m/s² upward, the force applied to the rope by the 5.0-kg mass is 20 N downward.

To learn more about force visit:

https://brainly.com/question/12785175

#SPJ4

a 62.0-kg woman runs up a 4.28-m high stairway in a time of 4.20 s. what average power did she supply?

Answers

The woman supplied an average power of 625 W while climbing the stairway.

How to find average power?

To find the average power the woman supplied, we need to use the formula:

average power = work done / time

The work done is equal to the change in potential energy of the woman as she climbs the stairs. The change in potential energy is given by:

ΔPE = mgh

where m is the mass of the woman, g is the acceleration due to gravity, and h is the height of the stairway.

So, ΔPE = (62.0 kg) x (9.81 m/s^2) x (4.28 m) = 2627 J

The time taken by the woman is 4.20 s.

Therefore, the average power she supplied is:

average power = work done / time = 2627 J / 4.20 s ≈ 625 W

Therefore, the woman supplied an average power of 625 W while climbing the stairway.

Learn more about average power

brainly.com/question/30888338

#SPJ11

what is the electric force of the molecule on the proton? express your answer with the appropriate units.

Answers

The electric force of the molecule on the proton is  N = kg m / [tex]s^2[/tex].

To calculate the electric force of a molecule on a proton, we need to know the charge of the molecule and the distance between the molecule and the proton.

If the molecule has a net charge of q, and the distance between the molecule and the proton is r, then the electric force between them is given by Coulomb's law:

F = kqq_proton / r^2

where k is the Coulomb constant (8.9875 x 10^9 N m^2/C^2), and q_proton is the charge of the proton (which is positive and has a magnitude of 1.6022 x 10^-19 C).

The units of the electric force depend on the units used for charge (C) and distance (m). If we express q in Coulombs, r in meters, and F in Newtons, then the units of the electric force can be written as:

N = ([tex]N m^2/C^2[/tex]) * C * C / [tex]m^2[/tex]

which simplifies to:

N = kg m / [tex]s^2[/tex]

Therefore, the units of the electric force are Newtons (N).

To learn more about Newtons visit:

https://brainly.com/question/3273157

#SPJ11

A parallelâplate capacitor is charged by connection to a battery. If the battery is disconnected and the separation between the plates is increased what will happen to the charge on the capacitor and the voltage across it?
A. Both remain fixed.
B. Both increase.
C. Both decrease.
D. The charge increases and the voltage decreases.
E. The charge remain fixed and the voltage increases.

Answers

If the battery is disconnected and the separation between the plates is increased then the  charge increases and the voltage decreases. Option D

When a parallel-plate capacitor is charged by connection to a battery, it stores electrical energy in the form of separated charges on its plates. If the battery is disconnected and the separation between the plates is increased, the charge on the capacitor remains the same, but the voltage across it decreases.
This is because the capacitance of the capacitor is determined by its geometry, which includes the distance between its plates. When the distance between the plates is increased, the capacitance decreases, and as a result, the voltage across the capacitor also decreases. This is because the charge stored on the plates is spread out over a larger area, reducing the electrical potential difference between them.
It is important to note that the charge on the capacitor remains the same because charges cannot be created or destroyed. They can only be moved around or redistributed. In this case, the charges on the capacitor are simply redistributed over a larger area.
Therefore, the correct answer to the question is option D: the charge increases and the voltage decreases. It is important to understand the relationship between capacitance, charge, and voltage in order to accurately predict the behavior of capacitors in different situations.

For more such questions on voltage visit:

https://brainly.com/question/1176850

#SPJ11

Consider three identical resistors, each of resistance R. The maximum power each can dissipate is P. Two of the resistors are connected in series, and a third is connected in parallel with these two. What is the maximum power this network can dissipate?
A. 2P/3
B. 2P
C. 3P
D. 3P/2
3P/2

Answers

The maximum power this network can dissipate will be 3P/2. The right answer is D.

When two identical resistors are connected in series, their total resistance is 2R.

When a third identical resistor is connected in parallel with these two, the total resistance of the circuit becomes R/3. Using the formula for power,

[tex]P = V^2/R[/tex],

we can calculate the maximum power that each resistor can dissipate as

[tex]P = V^2/R.[/tex]

When two resistors are connected in series, the voltage across them is divided equally.

Therefore, the voltage across each resistor in the series combination is V/2.

When a third resistor is connected in parallel with these two, the voltage across each of the series resistors remains the same (V/2), but the voltage across the parallel resistor is also V/2.

The total power dissipated in the series resistors is

[tex]P1 = (V/2)^2 \times R = V^2/4R.[/tex]

The total power dissipated in the parallel resistor is

[tex]P2 = (V/2)^2 \times (R/3) = V^2/12R.[/tex]

The total power dissipated in the network is the sum of P1 and P2, which is [tex]5V^2/12R[/tex] or 3P/2.

Therefore, the maximum power that this network can dissipate is 3P/2. The right option is D.

For more such answers on Resistance

https://brainly.com/question/17563681

#SPJ11

In the celestial sphere model, the Sun's position along the western horizon at sunset changes because

Answers

In the celestial sphere model, the Sun's position along the western horizon at sunset changes due to the Earth's rotation on its axis.

The celestial sphere is an imaginary sphere with the observer at its center. It is used to map the positions of celestial objects such as stars, planets, and the Sun.

The Sun appears to move across the sky during the day because of the Earth's rotation on its axis. As the Earth rotates, different parts of the Earth's surface are facing the Sun at different times.At sunset, the western horizon is the part of the Earth's surface that is turning away from the Sun, while the eastern horizon is turning towards it. This causes the Sun to appear to move downwards in the western sky until it disappears below the horizon. The exact position of the Sun along the western horizon at sunset changes throughout the year due to the tilt of the Earth's axis and its revolution around the Sun, which causes the apparent path of the Sun to shift north and south in the sky over the course of the year.

for such more questions on Earth's rotation

https://brainly.com/question/28308953

#SPJ11

a hot low density gas such as any the colorful nebulae imaged by the hubble space telescope emits a spectrum known as

Answers

A hot low density gas such as any the colorful nebulae imaged by the hubble space telescope emits a spectrum known as emission lines.

What is density?

Density is a physical property of matter that measures its mass per unit of volume. It is expressed in terms of kilograms per cubic meter (kg/m3) and is an important factor in determining an object's ability to float or sink in a liquid or a gas. Density is also used to calculate the specific gravity of a substance, which compares the densities of two different substances.

These lines are created when electrons in the gas are excited by photons and then transition to a lower energy state. The emission lines correspond to specific wavelengths of light and can be used to identify elements, as each element has a unique set of emission lines. This can be used to identify the elements present in the gas and gives an indication of its temperature and density.

The temperature of a hot low-density gas is usually lower than that of the surrounding environment. This is because the gas has fewer collisions with other particles and hence, the energy transfer from these collisions is lower. As the temperature of the gas decreases, its density also decreases. This is because the gas molecules have less kinetic energy, so they are less likely to collide with each other and form a denser structure.

The density of a hot low-density gas is usually lower than that of the surrounding environment. This is because the gas has fewer collisions with other particles and hence, the energy transfer from these collisions is lower. As the density of the gas decreases, its temperature also decreases. This is because the gas molecules have less kinetic energy, so they are less likely to collide with each other and form a denser structure.

To know more about density click-
https://brainly.com/question/1354972
#SPJ1

A boy takes a toy top and pulls on a string to make the top spin. The top can be considered a solid disk (I=½MR^2) and has a mass of 0.100kg and a radius of 0.0200m. The top starts from rest and ends up spinning at 15.0rev/s after 0.800s. What is the angular acceleration of the top?

Answers

The angular acceleration of the top is 117.81 rad/s^2.

To find the angular acceleration of the top, we can use the formula:
angular acceleration = (final angular velocity - initial angular velocity) / time

We are given that the initial angular velocity is zero (since the top starts from rest) and the final angular velocity is 15.0 rev/s. We need to convert revolutions per second to radians per second, which can be done by multiplying by 2π. So:

final angular velocity = 15.0 rev/s * 2π rad/rev = 94.25 rad/s
The time is given as 0.800 s. Now we can plug these values into the formula:
angular acceleration = (94.25 rad/s - 0 rad/s) / 0.800 s
angular acceleration = 117.81 rad/s^2

Therefore, the angular acceleration of the top is 117.81 rad/s^2.

To know more about angular acceleration, please click on:

https://brainly.com/question/30238727

#SPJ11

You are looking toward the north and see the Big Dipper to the right of Polaris. Fifteen minutes later, the Big Dipper will appear to have moved in roughly what direction?

Answers

The exact direction will depend on your location and the time of year, but in general, the stars appear to move approximately 15 degrees per hour

Assuming that you are in the Northern Hemisphere, Polaris (also known as the North Star) is located very close to the north celestial pole, which is the point in the sky around which the stars appear to rotate.

The Big Dipper is a well-known asterism that is part of the constellation Ursa Major, and it appears to circle around the north celestial pole over the course of the night.If you are looking toward the north and see the Big Dipper to the right of Polaris, this means that the Big Dipper is located to the east of Polaris. As the Earth rotates on its axis, the stars appear to move from east to west across the sky, with the stars located to the east of the meridian (an imaginary line running from due north to due south through the zenith) rising before the stars to the west of the meridian.

Therefore, fifteen minutes later, the Big Dipper will have moved to the west of its current position, which means that it will appear to have moved in the direction of the west.

for such more questions on Hemisphere

https://brainly.com/question/18994375

#SPJ11

T/F A single pulley will crease a larger mechanical advantage when lifting a weight

Answers

Multiple pulleys in combination can increase the mechanical advantage of the system.

False.

A pulley is a simple machine that changes the direction of the force applied to it. A single pulley can only change the direction of the force required to lift an object, and it does not increase the mechanical advantage of the system.

Mechanical advantage refers to the ratio of the output force to the input force in a system. It is a measure of the amount by which a machine can multiply the force applied to it. In a simple pulley system, the mechanical advantage is equal to 1 because the output force is the same as the input force.

However, the mechanical advantage can be increased by using multiple pulleys in combination, such as in a block and tackle system. In a block and tackle system, several pulleys are arranged in such a way that the rope or cable passes through them multiple times. This arrangement multiplies the mechanical advantage of the system, making it easier to lift heavy objects.

So, in summary, a single pulley does not increase the mechanical advantage when lifting a weight. It only changes the direction of the force applied to the system. Multiple pulleys in combination can increase the mechanical advantage of the system.

To learn more about mechanical visit:

https://brainly.com/question/20885658

#SPJ11

when a ball is hit with a given force, why does contact over a long time impart more speed to the ball than contact over a short time?

Answers

Answer:

Δ M V (change in momentum) = F Δt

The change in momentum (hence the change in velocity) is proportional

to the time that is applied

Calculate the % N in these common fertilizers:A. NH3B. NH4NO3

Answers

To calculate the % N in common fertilizers, we need to consider the percentage of nitrogen in each of the compounds.

A. NH3: NH3 contains one nitrogen atom and three hydrogen atoms. The atomic mass of nitrogen is 14.01 g/mol, while the molecular mass of NH3 is 17.03 g/mol. Therefore, the percentage of nitrogen in NH3 is:

(14.01 g/mol / 17.03 g/mol) x 100% = 82.1% N

B. NH4NO3: NH4NO3 contains two nitrogen atoms, four hydrogen atoms, and three oxygen atoms. The atomic mass of nitrogen is 14.01 g/mol, while the molecular mass of NH4NO3 is 80.04 g/mol. Therefore, the percentage of nitrogen in NH4NO3 is:

[(2 x 14.01 g/mol) / 80.04 g/mol] x 100% = 35.0% N

So, the % N in NH3 is 82.1%, and the % N in NH4NO3 is 35.0%.

To know more about fertilizer nitrogen % click this link-

brainly.com/question/13121539

#SPJ11

When a temporary threshold shift becomes a permanent threshold shift.

Answers

A temporary threshold shift (TTS) is a hearing loss that occurs after exposure to loud sounds or noise. This hearing loss is usually temporary and typically resolves within a few hours to a few days.

However, if the noise exposure is prolonged or the sound level is extremely high, a temporary threshold shift can become permanent.

Exposure to loud noise: When a person is exposed to loud noise or sounds, the hair cells in the inner ear can become damaged. This damage can cause a temporary reduction in hearing sensitivity, known as a temporary threshold shift.Recovery period: After the noise exposure ends, the hair cells can begin to recover and the hearing loss can gradually improve. If the noise exposure was not too severe, the hearing should return to normal within a few hours to a few days.Continued exposure: If the person continues to be exposed to loud noise or sounds before the hair cells have fully recovered, the temporary the should shift can become more severe and longer-lasting.Damage to hair cells: Prolonged or repeated exposure to loud noise can cause permanent damage to the hair cells in the inner ear. Over time, this damage can accumulate, leading to a permanent reduction in hearing sensitivity, known as a permanent threshold shift.Diagnosis: A permanent threshold shift is typically diagnosed through a hearing test, which measures the person's ability to hear sounds of different frequencies and volumes.Treatment: There is no cure for a permanent threshold shift, but hearing aids or cochlear implants may be recommended to improve communication and quality of life.

In summary, a temporary threshold shift can become a permanent threshold shift if the noise exposure is prolonged or the sound level is extremely high.

for such more questions on  permanent threshold shift.

https://brainly.com/question/11831553

#SPJ11

The length of a simple pendulum with a period on Earth of 6.0 seconds is most nearly:

Answers

The length of a simple pendulum with a period on Earth of 6.0 seconds is nearly 8.95 meters.



A simple pendulum can be described as a device where its point mass is attached to a light inextensible string and suspended from a fixed support. The vertical line passing through the fixed support is the mean position of a simple pendulum.

To calculate the length, we can use the formula for the period of a simple pendulum:

T = 2π√(L/g), where T is the period, L is the length, and g is the acceleration due to gravity (approximately 9.81 m/s² on Earth).

First, we'll rearrange the formula to solve for L:
L = (T² * g) / (4π²)

Now, plug in the given period (T = 6.0 seconds) and the value of g (9.81 m/s²):
L = (6.0² * 9.81) / (4 * π²)

Calculate the result:
L ≈ 8.95 meters

Learn more about simple pendulum:

https://brainly.com/question/26449711

#SPJ11

Assuming that the wave speed varies little when sound waves are traveling though a material that suddenly changes density by 10%, what percentage of the incident wave intensity is reflected?

Answers

The reflection coefficient is approximately 0.025, which means that 2.5% of the incident wave intensity is reflected.

The amount of energy that is reflected depends on the difference in acoustic impedance between the two materials. Acoustic impedance is the product of the density and the speed of sound in the material.

If the density of a material changes by 10%, then its acoustic impedance also changes by 10%. Assuming that the speed of sound does not change much at the boundary, the difference in acoustic impedance between the two materials is approximately 20%.The reflection coefficient is the ratio of the reflected intensity to the incident intensity of the sound wave. It depends on the difference in acoustic impedance between the two materials:

[tex]R = [(Z2 - Z1)/(Z2 + Z1)]^2[/tex]

where R is the reflection coefficient, Z1 is the acoustic impedance of the first material, and Z2 is the acoustic impedance of the second material.Since the density of the second material has changed by 10%, its acoustic impedance has changed by 10% as well. Therefore, the difference in acoustic impedance between the two materials is approximately 20%. If we assume that the speed of sound does not change much at the boundary, then the acoustic impedance of the first material remains approximately the same.

Substituting the values into the formula for the reflection coefficient, we get:

[tex]R = [(Z2 - Z1)/(Z2 + Z1)]^2[/tex]

[tex]R = [(1.1Z1 - Z1)/(1.1Z1 + Z1)]^2[/tex]

[tex]R = [(0.1Z1)/(2.1Z1)]^2[/tex]

[tex]R = 0.025[/tex]

for such more questions on reflection coefficient

https://brainly.com/question/31498600

#SPJ11

An automobile of mass 1500kg moving at 25.0m/s collides with a truck of mass 4500kg at rest. The bumpers of the two vehicles lock together during the crash. Compare the force exerted by the car on the truck with that exerted by the truck on the car during the collision.

Answers

The force exerted by the car on the truck is equal to 7500 N, and the force exerted by the truck on the car is equal to -7500 N.

According to Newton's third law of motion, for every action, there is an equal and opposite reaction. Therefore, during the collision between the car and the truck, the force exerted by the car on the truck is equal in magnitude but opposite in direction to the force exerted by the truck on the car.

To determine the force exerted by the car on the truck, we can use the formula for impulse, which is the change in momentum of an object over a given period of time:

Impulse = Force x Time = Δp

where Δp is the change in momentum of the object and t is the time interval during which the force is applied.

In this case, the bumpers of the car and truck lock together, which means that they move as a single unit after the collision. Therefore, we can apply the law of conservation of momentum, which states that the total momentum of an isolated system remains constant.

Initially, the car has a momentum of:

p_car_initial = m_car * v_car_initial = 1500 kg * 25.0 m/s = 37500 kg m/s

Since the truck is at rest initially, its momentum is zero:

p_truck_initial = 0

After the collision, the two vehicles move together as a single unit with a common final velocity. Let the final velocity of the car-truck system be v_final.

Using conservation of momentum:

p_car_initial + p_truck_initial = (m_car + m_truck) * v_final

37500 kg m/s + 0 = (1500 kg + 4500 kg) * v_final

v_final = 5.0 m/s

Therefore, the momentum of the car-truck system after the collision is:

p_final = (1500 kg + 4500 kg) * 5.0 m/s = 30000 kg m/s

The change in momentum of the car-truck system during the collision is:

Δp = p_final - p_car_initial = 30000 kg m/s - 37500 kg m/s = -7500 kg m/s

Since the time interval during which the force is applied is not given in the problem, we cannot calculate the exact values of the forces exerted by the car on the truck and the truck on the car during the collision. However, we know that the forces are equal in magnitude but opposite in direction according to Newton's third law of motion.

Therefore, the force exerted by the car on the truck is equal to 7500 N, and the force exerted by the truck on the car is equal to -7500 N.

To learn more about kinetic energy click:

brainly.com/question/26472013

#SPJ4

STT 10.6 A block with an initial kinetic energy of 4.0 J comes to rest after sliding 1.0 m. How far would the block slide if it had 8.0 J of initial kinetic energy?A 1.4 M B 2.0 MC 3.0 MD 4.0 M

Answers

Initially, the block has kinetic energy, which is converted into work done against friction to bring the block to rest. We can use equation for work done, W = Fd, where F is force of friction and d is distance traveled by the block. Therefore, answer is option B.

Since the force of friction is constant, we can use the equation W = Fd = -ΔK, where ΔK is the change in kinetic energy.

ΔK = -4.0 J, and d = 1.0 m.

Using this equation, we get Fd = 4.0 J, value of the force of friction.

For the second scenario, ΔK = -8.0 J.

Solving for d, we get d = 2.0 m.

Hence correct option is: B.

To know more about kinetic energy, here

brainly.com/question/26472013

#SPJ4

Two pith balls are charged by touching one to a glass rod that has been rubbed with a nylon cloth and the other to the cloth itself.How will the two pith ball react with one another?

Answers

When a glass rod is rubbed with a nylon cloth and then used to charge two pith balls, the balls become positively and negatively charged, respectively, and will attract each other due to the electrostatic force.


Step 1: Understand the charging process
When the glass rod is rubbed with the nylon cloth, it gains a positive charge due to the transfer of electrons from the glass to the cloth. The nylon cloth becomes negatively charged.


Step 2: Charging the pith balls
When one pith ball touches the charged glass rod, it gains a positive charge due to the transfer of electrons from the pith ball to the glass rod.

When the other pith ball touches the charged nylon cloth, it gains a negative charge due to the transfer of electrons from the cloth to the pith ball.


Step 3: Interaction between the charged pith balls
Since one pith ball is positively charged and the other is negatively charged, they will attract each other due to the electrostatic force acting between them. This is because opposite charges attract one another.

To know more about electrostatic force click here:

https://brainly.com/question/9774180

#SPJ11

A 5-mol ideal gas system undergoes an adiabatic free expansion (a rapid expansion into a vacuum), going from an initial volume of 10 L to a final volume of 20 L. How much work is done on the system during this adiabatic free expansion?

Answers

The work done on the system during an adiabatic free expansion is equal to the change in internal energy of the system.

Due to the adiabatic nature of the process, the change in the system's internal energy is equal to the change in the system's total energy, which is equal to the opposite of the change in potential energy.

As a result, the work accomplished is equal to the system's potential energy at the final volume less its potential energy at the initial volume. Since the system is perfect, PV, where P is the pressure and V is the volume, gives the system's potential energy.

Since P1 and P2 are the pressures before and after the expansion, respectively, the work done on the system is equal to −(20L×P2 - 10L×P1).

To learn more about potential energy visit:

https://brainly.com/question/1455245

#SPJ4

how to caluclate the minium angle a pendulum can be dropped from to just make it around the peg without the string going slack?

Answers

The minimum angle from which a pendulum can be dropped so that it just makes it around the peg without the string going slack can be calculated using the conservation of energy principle.

Let's assume that the length of the pendulum is L and the radius of the peg is r.

The potential energy of the pendulum at its maximum height (when it is just about to be released) is equal to its kinetic energy at its lowest point (when it just makes it around the peg without the string going slack).

Therefore, we can equate these two energies as follows:

mgh = (1/2)mv²

where m is the mass of the pendulum, g is the acceleration due to gravity, h is the height from which the pendulum is dropped, and v is the velocity of the pendulum at its lowest point.

Since the velocity of the pendulum at its lowest point is zero, we can simplify the above equation to:

h = (1/2)(v²/g)

Now, we can use the conservation of energy principle to determine the velocity of the pendulum at its lowest point. The total energy of the pendulum is constant and is equal to the sum of its potential energy (mgh) and its kinetic energy

(1/2)mv².

Therefore, we can write:

mgh = (1/2)mv²

Simplifying this equation, we get:

v² = 2gh

Substituting this value of v² into the equation for h, we get:

h = (1/2)(2gh/g)

h = h

This means that the height from which the pendulum is dropped does not depend on its mass. Therefore, the minimum angle from which the pendulum can be dropped so that it just makes it around the peg without the string going slack is given by:

θ = sin⁻¹(r/L)

where θ is the minimum angle in radians, r is the radius of the peg, and L is the length of the pendulum. To convert the angle from radians to degrees, we can use the formula:

θ(degrees) = (θ(radians) / 180)/π

where π is the mathematical constant pi.

For more such answers on the pendulum

https://brainly.com/question/26524032

#SPJ11


Does anyone know how to do it

Answers

Only the resistor connected in series which is resistor A will have the least current passing through it.

Which resistor is the current the smallest?

In this problem, we have some resistors that are connected in series and some in parallel.

The formula of resistors connected in series is given as;

R(series = R1 + R2 + R3 + ....+ Rn

The formula for resistors connected in parallel are

R(parallel) = 1/ R1 + 1/R2 + 1 / R3 +...+ 1/Rn

In this case, we have to assume that the voltage passing through the circuit is uniform and let's assume is 2V

V = IR

I = currentR = resistanceV = voltage

I = V/R

From the equation above, we can see that only the series resistance will have a small current passing through it.

Learn more on resistors here;

https://brainly.com/question/469388

#SPJ1

If F = 40 N and M = 2.0 kg, what is the magnitude of the acceleration of the suspended object? All surfaces are frictionless.
1) 1.2 m/s2
2) 2.0 m/s2
3) 1.5 m/s2
4) 2.5 m/s2
5) 5.6 m/s2

Answers

The magnitude of the acceleration of the suspended object is:

a = 20 m/s²

To find the magnitude of the acceleration of the suspended object, you can use Newton's second law of motion, which states that force (F) is equal to mass (M) times acceleration (a):

In this case, we are given that the force acting on the suspended object is 40 N, and the mass of the object is 2.0 kg. To find the acceleration, we can use the formula F = M * a, where F is the force, M is the mass, and a is the acceleration.

F = M * a

Given F = 40 N and M = 2.0 kg, you can solve for a:

40 N = 2.0 kg * a

Now, divide both sides by the mass (2.0 kg):

a = 40 N / 2.0 kg

a = 20 m/s²

However, none of the provided options match the calculated acceleration.

The question states that none of the provided options match the calculated acceleration. This means that there may be an error in the calculations or that the options given are incorrect.

To learn more about magnitude, refer below:

https://brainly.com/question/14452091

#SPJ11

[Show student response to predict question] Describe how increasing the stimulus frequency affected the force developed by the isolated whole skeletal muscle in this activity. How well did the results compare with your prediction?

Answers

In this activity, increasing the stimulus frequency had a direct impact on the force developed by the isolated whole skeletal muscle. As the frequency of the stimulus increased, the muscle experienced a higher rate of nerve impulses. This led to a greater number of muscle fibers being activated, resulting in an increased force of contraction.

At lower frequencies, the muscle had sufficient time to relax between stimuli, allowing for individual twitches to be distinguished. However, as the frequency increased, the time between stimuli decreased, and the muscle could not fully relax. This caused summation, where the force of the subsequent contractions added up, resulting in a stronger muscle contraction overall.

Eventually, the stimulus frequency reached a point where the muscle contractions fused together, leading to tetanus – a sustained, maximal force contraction. This is the point at which the muscle developed its greatest force in response to the increasing stimulus frequency.

The results from this activity may have aligned with your prediction if you understood the relationship between stimulus frequency and muscle force. As frequency increases, so does the force generated by the muscle, up to the point of tetanus. Overall, the experiment demonstrated the essential role of stimulus frequency in modulating the force developed by whole skeletal muscles.

For more such questions on Stimulus frequency.

https://brainly.com/question/15908931#

#SPJ11

for an inelastic collision, which of the following statements are true for a system that includes two colliding objects? choose all that apply. for an inelastic collision, which of the following statements are true for a system that includes two colliding objects?choose all that apply. kinetic energy is lost. momentum is constant. kinetic energy is gained. momentum is lost. kinetic energy is constant. momentum is gained. submitprevious answersrequest answer incorrect; try again your answer indicates that you need to review the concepts associated with collisions. provide feedbacknext incorrect. incorrect; try again. feedback. your answer indicates that you need to review the concepts associated with collisions. end of feedback.

Answers

1. For an inelastic collision involving two colliding objects, the following statements are true: 1. Kinetic energy is lost. 2. Momentum is constant.

2. The total momentum of the system remains constant before

and after the collision, as dictated by the conservation of momentum.

1. The following claims are true for an inelastic collision between two objects:

Kinetic energy is lost

Momentum is conserved

Kinetic energy is not gained

Momentum is not lost

Kinetic energy is not constant

Momentum is not gained

Therefore, the correct statements are:

Kinetic energy is lost.

Momentum is conserved.

2. In an inelastic collision, the objects stick together after the collision,

and some of the initial kinetic energy is converted into other forms of

energy, such as heat or deformation.

However, The conservation of momentum requires that the total

momentum of the system be unchanged before and after the collision.

for such more question on inelastic collision

https://brainly.com/question/30055517

#SPJ11

a heavy block is suspended from a vertical spring. the elastic potential energy is stored in the spring is 0.8 j. what is the elongation of the spring if the spring constant is 100 n/m?

Answers

The elongation of the spring is approximately 0.126 meters.

To find the elongation of the spring when the elastic potential energy stored in the spring is 0.8 J and the spring constant is 100 N/m, we will use the formula for elastic potential energy:
E = (1/2) * k * x^2
where E is the elastic potential energy, k is the spring constant, and x is the elongation of the spring.

E = 0.8 J
k = 100 N/m

Rearrange the formula to solve for x:
x^2 = (2 * E) / k

Plug in the values:
x^2 = (2 * 0.8 J) / 100 N/m

Calculate the elongation (x):
x = √(1.6 / 100) = √0.016

x ≈ 0.126 meters

So, the elongation of the spring is approximately 0.126 meters.

Learn more about "spring": https://brainly.com/question/14670501

#SPJ11

What is the change in entropy (DS) when one mole of silver (108 g) is completely melted at 961°C? (The heat of fusion of silver is 8.82 ´ 104 J/kg.)

Answers

The change in entropy (DS) of the given one mole of silver is 76.18 J/K.

Mass of the silver, m = 108 g = 108 x 10⁻³kg

Temperature of melting, T = 961°C = 1204 K

Heat of fusion of silver, Q = 8.82 x 10⁴ J/kg

Change in entropy,

ΔS = Q/T

ΔS = (8.82 x 10⁴ x 104)/1204

ΔS = 76.18 J/K

To learn more about entropy change, click:

https://brainly.com/question/1301642

#SPJ4

T/F If you increase the time over which a force is applied, you can decrease the magnitude of the force and get the same change in momentum

Answers

True . If you increase the time over which a force is applied, you can decrease the magnitude of the force and still achieve the same change in momentum.

This concept is related to the Impulse-Momentum Theorem, which states that the impulse (force multiplied by time) is equal to the change in momentum.

To break it down step-by-step:
1. Impulse (I) is calculated as Force (F) multiplied by Time (t): I = F * t
2. Change in momentum (Δp) is equal to the impulse: Δp = I
3. When you increase the time (t) while maintaining the same change in momentum (Δp), you can decrease the force (F) since their product remains constant.

So, by increasing the time over which a force is applied, you can decrease the magnitude of the force while still achieving the same change in momentum.

To know more about Impulse-Momentum Theorem:

https://brainly.com/question/14121529

#SPJ11

Other Questions
With a standard diagnostic imaging instrumentation, which has the higher numerical value ?a. axial resolutionb. lateral resolutionc. neither, they have identical values FILL IN THE BLANK. the _______ into the future you want to forecast, the ____ accurate you are likely to be; why is this? How long does OARRS data need to be kept? A political party received an average of 34% support in recent polls plus or minus 3.4%, 19 times out of 20. Two subsequent polls showed 38% support and 27% support. How would you report on the meaning of these polls to the party membership? Which statement best supports the idea the father was glad his family did not move to Texas?Autobiography of Z. S. Hastings: Written for His Boysby Z. S. Hastings (excerpt)A. "[Father] changed his mind and sold his Texas land for a song in the shape of a beautiful colt. This colt grew into one of the prettiest and best horses your grandfather ever had." (paragraph 6)B. "At this time Daviers County was a wilderness of brush, trees and swamps, with plenty of wild game,-deer, coons, opossums, squirrels, turkeys, ducks, quails, snowbirds..." (paragraph 8)C. "In 1842 when I was about four years old my parents sold out and moved down the river five or six miles and bought a new, larger and better farm..." (paragraph 2)D. "If we had gone to Texas, boys, I do not know what might have been but I do know now that you are and that you have one of the best mothers that lives." (paragraph 7) (1 point) Find the sum of the following geometric series: 1 - 5 + 25 - 125 + ... +9.76562 x 106 Sum: g what polysaccaride would not exhibit the following type of linkage for all or a major portion of its structure? a. glycogen b. amylose c. cellulose d. amylopectin How can we indicate that some notes are no longer new to us? Using Clark's Rule, what is the dose of drug for an eight-year-old boy weighing 50 pounds? (The normal adult dose is 300 mg.) In Problem 5, suppose that Grinch and Grubb go into the wine business in a small country where wine is difficult to grow. The demand for wine is given by p = $480 - .2Q, where p is the price and Q is the total quantity sold. The industry consists of just the two Cournot duopolists, Grinch and Grubb. Imports are prohibited. Grinch has constant marginal costs of $6 and Grubb has marginal costs of $45. How much Grinch's output in equilibrium? a) 1,350 b) 2,025 c) 337.50 d) 675 Find the value of the following using suitable identity-125 729 8 How were groups that weren't considered white depicted in media? AlbeUnit Test12Active35 68Mark this and return9 10Read this excerpt from "Look Homeward, Angel."And whatever he touched in that rich fortress of his soul sprang into golden life: as the years passed, the fruittrees-the peach, the plum, the cherry, the apple-grew great and bent beneath their clusters. His grape vinesthickened into brawny ropes of brown and coiled down the high wire fences of his lot, and hung in a densefabric, upon his trellises, roping his domain twice around. They climbed the porch end of the house andframed the upper windows in thick bowers. And the flowers grew in rioting glory in his yard-the velvet-leaved nasturtium, slashed with a hundred tawny dyes, the rose, the snowball, the redcupped tulip, and thelily.The author uses sensory details in this excerpt to create images ofexcess and riches, to suggest interest in materialism.bountiful blooms and harvests, to suggest agricultural success.colorful visions and sceneries, to suggest artistic aptitude.climbing vines and fruit, to suggest time standing still.TIME REMAINING01:56:50NextSudanit The apses of the most expensive early Christian basilicas were decorated withA. FrescoB. egg temperaC. mosaicD. oil A local hamburger shop sold a combined total of 822 hamburgers and cheeseburgers on Tuesday. There were 72 more cheeseburgers sold than hamburgers. How many hamburgers were sold on Tuesday? Sam owns a hardware store, which sells nearly everything.Taylor comes in looking for some paint that will cover the brick faade around his fireplace. He explains to Sam that the brick can get warm when a fire is blazing but also it can get very cold when there is no fire and the temperatures outside are below freezing. Taylor asks him what is the best paint for this type of work. Sam takes him over to a certain aisle and points to a certain brand and says, "This paint is great for masonry, brick, and stone work inside and out."Melissa comes into the store right after Taylor and Sam greets her with a "how is the project coming along?" Melissa answers, "Oh, you mean the one where I am restoring some of my grandmothers porcelain pieces?" Sam replies, "Yes, that one." "Im just here to get some porcelain glue for this kind of work." Melissa goes off down the glue and adhesive aisle that Sam points her toward. In addition to purchasing the glue, Melissa also purchases some Christmas lights on the "as is" price table. When she gets home, the lights are not very bright, and one of the lights dim off and on occasionally.Sam checks Melissa out at the cash register, examining the tube of glue she bought. He doesnt say a thing other than, "Youre going to use this for porcelain?" Melissa responds, "Yes." Sam does not respond.Two days later, both Taylor and Melissa come back complaining that the paint and glue failed miserably. Taylor said that as the paint dried, it chipped off the wall. Melissa said that her porcelain pieces fell apart and crashed to the ground destroying several valuable pieces. Plus, she wants a refund for her Christmas lights.Check All That Apply:a. Taylor sues Sam in small claims court, arguing that Sam breached an express warranty. Sam claims that it is not his fault that the paint was not the correct paint for the job at hand, and Taylor should have carefully read all the labels before choosing. The court finds for Taylor. Is this a correct decision and is this statement by Sam an expressed warranty?b. Melissa also sues. She claims that Sam violated his express warranty as well, as evidenced by their conversation. Is this argument legitimate?c. Melissa wants a refund on the purchase of the Christmas lights. Sam refuses, claiming that his "as is" sign is a clear disclaimer of implied warranties. Compare- Reread lines 23-26, what does the authors description of the characters actions reveal about the real-life working conditions at the factory? How does this description of the working conditions compare to the description in lines 2130 of the excerpt from Flesh & Blood So Cheap? define fulcrum. Application of force at some distance from the fulcrum creates ____________. Distance between applied force and fulcrum is the________. What is the equation that relates all of these /over the last several decades, the composition of the democratic and republican parties has changed in important ways. a major partisan shift has occurred in the south, but other demographic changes have also been identified. changes in party composition are reflected at different rates in presidential elections than in congressional elections. a) identify one specific trend evident in the figure above. b) choose two of the following and use each to explain why southern voters from 1948 to 2000 were electing democratic candidates to congress more frequently than choosing democratic candidates for the presidency. - incumbency advantage - gerrymandering - differences between state and national parties c) several other changes in party composition have emerged in the past few decades. select three of the following groups and for each explain how parties have changed in composition with respect to that group. - catholics - labor union members - women - social conservatives Which quantitative statistics should be used. Explain your choice using complete sentences. Options include: Pearson correlation, independent t-test, paired sample t-test, analysis of variance (ANOVA)A study aimed to examine the relationship between fasting and academic performance. The predictor variable was fasting students, and the criterion variable was cognitive functioning using the Cambridge Neuropsychological Test Automated Battery (CNTAB). Participants were 30 healthy men (n=15) and women (n=15) between 18 and 23.